Problemas resueltos ecuaciones diferenciales PDF

Title Problemas resueltos ecuaciones diferenciales
Pages 9
File Size 382.5 KB
File Type PDF
Total Downloads 227
Total Views 354

Summary

AQUÍ TIENE ALGUNOS PROBLEMAS RESUELTOS DE LA GUIA 4 I-Encuentre el diferencial total para: 2) w  2z 3 y sen x Solución: Sabemos que si z=f(x,y) es una función de dos variables el diferencial total dz es z z dz  dx  dy x y Pero si w=f(x,y,z) el diferencial total de w es: w w w dw  dx  dy ...


Description

Accelerat ing t he world's research.

Problemas resueltos ecuaciones diferenciales Erick Jonathan Trigueros Portillo

Related papers Calculo Vect orial Henry Huaraca

Cálculo Avanzado Jose Mart inez Calculo Avanzado USACH Pedro Jesus Jarquin Valencia

Download a PDF Pack of t he best relat ed papers 

AQUÍ TIENE ALGUNOS PROBLEMAS RESUELTOS DE LA GUIA 4 I-Encuentre el diferencial total para:

2) w  2z 3 y sen x

Solución: Sabemos que si z=f(x,y) es una función de dos variables el diferencial total dz es

dz 

z z dx  dy y x

w w w dz dy  dx  z y x

Pero si w=f(x,y,z) el diferencial total de w es:

dw 

Entonces resolviendo el problema 2 se tiene: w w w  2z 3 y Cosx ;  6z 2 y Senx  2z 3 Senx ; x z y Sustituyendo en

dw 

w w w dz , se tendrá: dy  dx  z y x

R// dw  2z 3 y Cosxdx  2z 3 Senxdy  6z 2 y Senxdz II-Usando diferenciales, resuelva el siguiente problema. Resolvamos el problema 9 9) Las medidas tomadas al radio de la base y a la altura de un cono circular recto son 10cm y 25cm, respectivamente, con una tolerancia de error posible de 0.1cm en cada caso. Estime el error máximo en el volumen del cono. Solución: Sabemos que el volumen del cono viene dado por: 1 V  πr 2h ; 3 Nota: Dibuje un cono y colóquele las dimensiones especificadas y para los otros problemas conviene hacer siempre una representación grafica. Entonces el diferencial total de volumen del cono viene dado por: v v dv  dr  dh r h v 2 v 1  πrh  π r2 ; r 3 h 3 Sustituyendo en dv  dv 

2

π r hdr 

1

v r

dr 

v

h

dh se tiene:

π r 2 dh ,

3 Como r = 10 cm. , h = 25 cm. 3

y

dr = dh = 0.1 cm.

1

Entonces: dv 

2 3

π(10 cm )(25 cm )(0.1cm) 

dv  16.7 π cm 3  3.3 π cm 3

1 3

π(10cm)2 (0.1 cm)

dv  20 π cm 3

dv  62.8318 cm 3

Por lo tanto el error máximo en el volumen del cono es.

R// dv  62.8318 cm3

III. Usando diferenciales, encontrar la variación aproximada de la función dada, cuando las variables experimentan los cambios que se indican. 13) f x, y  x2  2xy  3y , cuando x,y  cambia de 2,3 a 2.1,3.2

Solución: Como z  f x, y  x 2  2xy  3y Entonces el diferencial total de la función viene dado por: z z dy dx  dz  df  y x z

x

 2x - 2y

;

Sustituyendo en dz 

z

z

x

y

 -2x  3

dx 

z

y

dy se tiene:

dz  (2x - 2y)dx  (-2x  3)dy ,

Como x = 2 , y = 3

entonces dx = 2.1-2 dx = 0.1

; dy = 3.2-3 ; dy = 0.2

Luego dz  (2(2) - 2(3))(0.1)  (-2(2)  3)(0.2) dz  -0.2 - 0.2 dz  -0.4

R//Por lo tanto variación aproximada de la función dada, cuando x,y  cambia de

2,3

a 2.1,3.2 es:

dz  -0.4

2

IV. Encuentre

dw , mediante la regla de la cadena y luego exprese a w como función de dt

t, y vuelva a derivar. Compare los resultados. Resolvamos el problema 21 21) w 

x2  y2  z2

x  2t , y  4t , z  6t

;

Al revisar el material se tiene que: dw dt



w dx



x dt

Como w 

w dy y dt



w dz z dt

x 2  y 2  z 2 entonces:

w

w  x 2  y 2  z 2  w  (x 2  y 2  z 2 )1 2 



1 2 (x  y 2  z2 ) 1 2 (2x) 2

x w   x w

w  x 2  y 2  z 2  w  (x 2  y 2  z 2 )1 2 

y



w y

w

w  x 2  y 2  z 2  w  (x 2  y 2  z 2 )1 2 





x

x 2  y 2  z2

1 2 (x  y 2  z2 ) 1 2 (2y) 2



y

x 2  y 2  z2

1 2 (x  y 2  z2 ) 1 2 (2z) 2

z w   z

También: x  2t 

dx dt

2

Sustituyendo en:

dw dt dw dt

 

x

x 2  y 2  z2 2x

y  4t 

;

x 2  y 2  z2

dw dt



(2)  

w dx x dt

dy dt



4

w dy y dt

y

x 2  y 2  z2 4y

x 2  y 2  z2



; 

z

x 2  y 2  z2

z  6t 

w dz z dt

(4) 

dz dt

,

6

se tiene:

z

x2  y2  x2 6z

x2  y2  x2

Sustituyendo a x, y, z, se tiene:

3

(6)

dw dt

dw dt

dw dt







2(2t)

(2t)2  (4t)2  (6t)2

4t

4t 2  16t2  36t2 56t





4(4t)

(2t)2  (4t)2  (6t)2

16t

4t 2  16t2  36t2





6(6t)

(2t)2  (4t)2  (6t)2

36t

4t 2  16t2  36t2

56t2

Por lo tanto: dw dt dw dt

 

56t 56t2 56



56t 56t

Racionalizando

56

 56 dt Ahora expresemos a w como función de t, y volvamos a derivar, pero esta vez en forma ordinaria. (No parcialmente) R//

w  x2  y2  z2

dw

;

x  2t , y  4t , z  6t

Sustituyendo:

w  (2t)2  (4t)2  (6t)2  w  4t 2  16t 2  36t 2  w  56t 2 Derivando ordinariamente se tiene: dw 1 w  (56t2 )1 2   (56t2 ) 1 2 (112t) dt 2 dw 1 dw 56t dw 56t  (56t2 ) 1 2 (112t)     dt 2 dt dt 56t 56t2 dw 56  dt 56

Simplificando:

 56 dt Nota: Compare que las respuestas son iguales. A este tipo de derivada se le llama DERIVADA TOTAL.

R//

dw

4

V-Use la regla de la cadena de la manera más adecuada para calcular la derivada parcial w, respecto r, θ , s y t, según el caso.

y ; x  r cosθ , y  r senθ x w w y Para este caso queremos encontrar θ r

Resolvamos el problema 27) w  arc tg

Usando el material de la clase se tiene: Si w = f(x,y) es una función derivable de x y de y, donde x = g(r, θ ), y =h(r, θ ) y las derivadas parciales de g y h existen, entonces: w 



w x x 



w y y 

w r

;



w x x r



w y y r

Calculando las derivadas parciales por separado: y Como w  arc tg x y x w w x  rSenθ ;  ;  θ x x 2  y 2 y x 2  y 2 w x



y ; x2  y2

x r

 Cosθ

Sustituyendo las derivadas en: w θ



;

w y

x

x2  y2 w w x w y   x  y  

  y  x   rSenθ     x2  y2   x2  y2   





; ;

y

θ

 rCosθ

y r

 Senθ

 rCosθ   

Sustituyendo también x  r cosθ , y  r senθ



- rSen  (rCos )2  (rSen )2 

w



w



θ

θ

w θ



   rCos  rSenθ    rCosθ    (rCos )2  (rSen )2    

r 2 (Sen ) 2  r 2 (Cos ) 2  r 2 (Sen )2





   r 2 (Cos ) 2     r 2 (Cos )2  r 2 (Sen )2    

r 2 (Cos ) 2 r 2 (Sen ) 2  2 2 2 r 2 ((Cos ) 2  (Sen )2 ) r ((Cos )  (Sen ) )

Pero (Cos )2  (Sen )2  1, entonces

5

w θ

w θ





2 2 r 2 Senθ 2 r Cosθ   r 2 (1) r 2 (1)

r 2  Senθ 2  Cosθ 2 



θ

θ



r 2 Senθ 2  r 2 Cosθ 2 r2

  w  Senθ 2  Cosθ 2

r2

w

R//

w



θ

1

Calculemos ahora w r



w x x r



w y y r

w r

Sustituyendo las derivadas en: w x

w x

w r





w r

y ; 2 2 x y

y ; x2  y2



x

θ

x r

 rSenθ

 Cosθ

;

  y  x  Cosθ     x2  y2   x2  y2   



;



w x x r

w y

w y







w y y r

x

x2  y2

x

x2  y2

;

θ

;

 Senθ   

Sustituyendo también x  r cosθ , y  r senθ w θ

R//



w 0 - rSen Cosθ  rCos Senθ   2 2 2 θ (rCos )  (rSen ) 2 (rCos )  (rSen )

w θ

0

6

y

 rCosθ

y r

 Senθ

VI. Calcule

z z y , usando derivación parcial implícita para cada uno de los siguientes x y





casos. Resolvamos el problema 35) xln 2y  z3  9 Para este caso debemos calcular

Zx y Zy

Usando el material de clase tenemos: Si f(x,y,z)=0, y si definimos a z como función de x , y , es decir: z=f(x, y), entonces: f f fy y z z fx ; ,   x     f f fz fz x y z z Como xln 2y  z3  9 , tenemos que igualar a cero xln 2y  z 3  9  0











Luego si: f(x, y, z)  xln 2y  z 3  9 f ln(2y  z 3 ) Z   x  x x f (3z2 ) z (2y  z 3 )

R//

(2y  z 3 )ln(2y  z 3 ) Z  x 3xz2

2x f y (2y  z 3 ) Calculando la otra parcial tenemos: Z    y f 3xz2 z (2y  z 3 ) Z

Al simplificar

R//

y



Zy  

2 x(2 y  z 3 ) 3 xz 2 (2 y  z 3 )

2 3z 2

7



VII. Usando regla de la cadena para funciones de varias variables resuelva los siguientes problemas: Resolvamos el problema 40 Utilice la ley de un gas ideal (PV=KT), con k= 0.8, para obtener la tasa a la que la temperatura varia en el instante en el que el volumen del gas es de 15 litros y el gas está bajo una presión de 12 atm si el volumen se incrementa a una tasa de 0.1 litro / min. y la presión disminuye a la tasa de 0.2 atm/min. Solución: Para resolver este tipo de problemas, se debe hacer una representación gráfica del problema, de la función despejar la variable requerida y considerar que todas las funciones dependen de la variable tiempo t. T

PV K

;

donde P y V dependen

Usando la regla de la cadena se tiene:

de dT dt

t,



K

T dV V dt



una constante.

T dP P dt

Pero del problema se tiene: K=0.8, Además: dV dt

V=15 litros, P=12 atm.

 0.1 litro / min

dP dt

 0.2

atm / min

Calculando las derivadas: dT P dV V dP   dt K dt K dt Sustituyendo: dT dt dT dt

 12   15  0.1     0.2  0.8   0.8 



 150.1   18.75 0.2





 2.25 dt Lo que debe interpretarse que cuando la presión disminuye a la tasa de 0.2 atm/min. y el volumen se incrementa en 0.1 litro / min., la temperatura disminuye a razón de 2.25 grados/min. En estos problemas cuando se sustituye es conveniente no usar unidades, ya que las unidades de la constante K dependerán de las otras variables y en este caso las unidades de K no nos las dan.

R//

dT

8...


Similar Free PDFs